Nicht integrierbare Wellenfunktionen

Angenommen, eine Störung erster Ordnung ergibt eine glaubwürdige Korrektur der Energie, aber eine Korrektur der Wellenfunktion, die nicht quadratintegrierbar ist. Das kann passieren, ich sehe keinen Grund, warum es nicht könnte. Es sei denn, es gibt Beweise dafür, dass es nicht passieren kann (ich habe keine gefunden). Und wenn es passiert, würden Sie dann der Energieverschiebung glauben, die Sie berechnet haben?

Auf den Kopf gestellt, das sollte das Kato-Rellich-Theorem verbieten. Aber ich werde es sicherheitshalber prüfen.
Ja, danke ... Der Punkt ist, dass in der Störungstheorie die Korrektur 1. Ordnung der Eigenfunktion als Linearkombination aller ungestörten Eigenfunktionen ausgedrückt wird. Jede der ungestörten Eigenfunktionen ist natürlich quadratintegrierbar, aber die Reihe kann eine nicht integrierbare Funktion ergeben.
Die Korrekturen selbst hängen von Ihrem Potenzial ab. Die Frage ist dann, welches Potential Sie Ihrem Problem hinzufügen lassen. Am Beispiel des Wasserstoffatoms erhalten wir eine Korrektur, die sich wie folgt verhält Σ m 2 n 2 m 2 n 2 < n ( 0 ) | v | m ( 0 ) > . Wenn wir ein Potential einführen können, dessen Matrixelemente langsamer abklingen als n 2 m 2 , können wir eine divergierende Reihe erhalten.
Das ist ein heikles Problem, es sollte in Kapitel VI von Kato, Perturbation Theory for Linear Operators, Springer 1966, diskutiert werden
Ich bin neugierig zu wissen, in welchem ​​Kontext dies geschieht.
Nicht genau das, was Sie fragen, aber betrachten Sie den harmonischen Oszillator Hamiltonian H mit einer λ x 4 Störung. Dies hat "glaubwürdige" Korrekturen für die Eigenwerte und Eigenvektoren, aber für λ < 0 , H + λ x 4 hat keine diskreten Eigenwerte, daher können Sie formalen Störungsberechnungen nicht glauben.
erinnert mich an 0celo7s ähnliche, aber umfassendere Frage, ob „mathematische Nachlässigkeit in QM jemals zu falschen Vorhersagen führt“ physical.stackexchange.com/questions/348913/…

Antworten (2)

Ich denke, der Wiki-Beweis für den Korrekturbegriff erster Ordnung ist ziemlich klar. https://en.wikipedia.org/wiki/Perturbation_theory_(quantum_mechanics)#First_order_corrections

Der wichtige Satz, der mit Ihrer Frage verbunden ist, lautet "Let v ein Hamiltonoperator sein, der eine schwache physikalische Störung darstellt". Der Kern der Störung ist die Linearisierung unter kleinen Änderungen, also denken Sie nicht v Wie einige übliche Potentiale, die Singularitäten enthalten, bedeutet dies wahrscheinlich, dass Ihr System durch eine unendliche Wand auf einen viel kleineren Bereich beschränkt ist.

Dann, bevor Sie den Korrekturterm erhalten, gibt es einen Ausdruck ( E n ( 0 ) E k ( 0 ) ) k ( 0 ) | n ( 1 ) = k ( 0 ) | v | n ( 1 ) was garantierte, dass die RHS eine kleine endliche Zahl ist, und jeder peatubierte Basisvektor (zumindest der erste) ist quadratisch integrierbar. (quadratisch integrierbar bedeutet, dass Sie eine endliche Zahl erhalten können, nicht Riemann-Integrierbarkeit.)

Wenn Sie eine konstruiert haben v so dass es nicht quadratisch integrierbar ist, sollten Sie keine Störungstheorie verwenden, da dies ein "Kick" ist, keine Störung.

Nach den Grundlagen der QM sind die einzigen Wellenfunktionen, die quadratisch integrierbar sind, physikalische. Daher können die Wellenfunktionen , die nicht quadratisch integrierbar sind, nur in der Mathematik auftreten, aber kein physikalisches System kann eine solche Wellenfunktion haben .

Neben dem ursprünglichen Hamilton -Operator hängt die Korrektur von Energien und Wellenfunktionen nur von den Störungen ab ( Wechselwirkungs-Hamilton -Operator ). Indem wir uns auf physikalisch akzeptable Wellenfunktionen beschränken, können wir die zulässigen Störungen auf das gegebene ursprüngliche System im Wesentlichen einschränken.

Das Tolle ist, dass diese Art von Beschränkungen für erlaubte Wechselwirkungen natürlicherweise im Kontext der Quantenfeldtheorie (QFT) vorkommen . Durch Aufrufen der Gauge-Invarianz auf dem ursprünglichen Lagrangian .

Zum Beispiel erhalten wir durch Aufrufen der Gauge-Invarianz auf dem freien Dirac-Lagrangian die Quantenelektrodynamik , QED , Lagrangian , die nur einen Wechselwirkungsterm haben, der der Fermion-Photon-Wechselwirkung entspricht.

Auch in der QFT können nicht-physikalische Wechselwirkungen durch Dimensionsanalyse und Renormierbarkeit der Theorie ausgeschlossen werden.

Also keine freien Partikel für Sie?
@lalala was meinst du?
Eigenfunktionen des Hamilton-Operators für freie Teilchen sind nicht quadratisch integrierbar.
@lalala Nein. Es ist quadratisch integrierbar. Für ein freies Teilchen gilt: \psi_k(r) = [e^{ik.r}]/\sqrt{V} und \integral dr |\psi_k(r)|^2 = 1.
Es ist seitdem nicht integrierbar ψ ψ ist ein Konstante, also ergibt die Integration über den gesamten Raum Unendlichkeiten ... es sei denn natürlich v ist auch unendlich, aber dann dein ψ k ( r ) ist überall null...